15

𝛇(𝟐)=𝛑²/𝟔 の証明

557
0
$$\newcommand{BA}[0]{\begin{align*}} \newcommand{BE}[0]{\begin{equation}} \newcommand{bl}[0]{\boldsymbol} \newcommand{D}[0]{\displaystyle} \newcommand{EA}[0]{\end{align*}} \newcommand{EE}[0]{\end{equation}} \newcommand{h}[0]{\boldsymbol{h}} \newcommand{k}[0]{\boldsymbol{k}} \newcommand{L}[0]{\left} \newcommand{l}[0]{\boldsymbol{l}} \newcommand{m}[0]{\boldsymbol{m}} \newcommand{n}[0]{\boldsymbol{n}} \newcommand{R}[0]{\right} $$
$\D \bf Theorem.$
$\D\zeta(2)=\frac{\pi^2}{6}$

$\D \bf Lemma.$
$\D\zeta(2)=\frac{4}{3}\sum_{n=0}^\infty \frac{1}{(2n+1)^2}=2\sum_{n=1}^\infty \frac{(-1)^{n-1}}{n^2}$

$Proof~1.$
$\BA\D \zeta(2)&=\frac{4}{3}\sum_{n=0}^\infty \frac{1}{(2n+1)^2}\\ &=\frac{4}{3}\int_0^1 \frac{\ln\frac{1}{x}}{1-x^2}\,dx\\ &=\frac{4}{3}\int_0^1\int_0^\infty \frac{t}{(1+t^2)(1+t^2x^2)}\,dt\,dx\\ &=\frac{4}{3}\int_0^1 \int_0^\infty \frac{1}{(1+t^2)(1+x^2)}\,dt\,dx\\ &=\frac{4}{3}\frac{\pi}{4}\frac{\pi}{2}\\ &=\frac{\pi^2}{6} \EA$

$Proof~2.$
$\BA\D \zeta(2)&=\frac{4}{3}\sum_{n=0}^\infty \frac{1}{(2n+1)^2}\\ &=\frac{4}{3}\sum_{n=0}^\infty \frac{\binom{2n}{n}}{2^{2n}(2n+1)}\int_0^1 \frac{x^{2n+1}}{\sqrt{1-x^2}}\,dx\\ &=\frac{4}{3}\int_0^1 \frac{\sin^{-1}x}{\sqrt{1-x^2}}\,dx\\ &=\frac{2}{3}(\sin^{-1}1)^2\\ &=\frac{\pi^2}{6} \EA$

$Proof~3.$
$\quad\psi'(z)$の相反公式
$\BA\D \psi'(z)+\psi'(1-z)=\frac{\pi^2}{\sin^2\pi z} \EA$

より

$\BA\D \pi^2&=2\psi'\L(\frac{1}{2}\R)\\ &=2\sum_{n=0}^\infty \frac{1}{\L(n+\frac{1}{2}\R)^2}\\ &=6\zeta(2) \EA$

$Proof~4.$
$\BA\D \sum_{k=1}^n \frac{1}{\tan^2\frac{(2k-1)\pi}{4n}}=2n^2-n \EA$


$\BA\D \frac{1}{x^2}-\frac{2}{3}<\frac{1}{\tan^2 x}<\frac{1}{x^2} \EA$

より
$\BA\D \sum_{k=1}^n\L(\L(\frac{4n}{(2k-1)\pi}\R)^2-\frac{2}{3}\R)<2n^2-n<\sum_{k=1}^n\L(\frac{4n}{(2k-1)\pi}\R)^2 \EA$

すなわち
$\BA\D \frac{\pi^2}{8}\L(1-\frac{1}{2n}\R)<\sum_{k=1}^n\frac{1}{(2k-1)^2}<\frac{\pi^2}{8}\L(1-\frac{1}{6n} \R) \EA$

となり,$n\to\infty$とすれば
$\BA\D \sum_{k=1}^\infty \frac{1}{(2k-1)^2}=\frac{\pi^2}{8} \EA$

$Proof~5.$
$\quad\rm Fourier~series$
$\BA\D \sum_{n=1}^\infty \frac{\sin nx}{n}=\frac{\pi-x}{2} \EA$

の両辺を$(0,\pi)$で定積分すると
左辺は

$\BA\D \sum_{n=1}^\infty \frac{1}{n}\int_0^\pi \sin nx\,dx=\sum_{n=1}^\infty \frac{1-(-1)^n}{n^2}=\sum_{n=1}^\infty \frac{2}{(2n-1)^2}=\frac{3}{2}\zeta(2) \EA$

右辺は
$\BA\D \int_0^\pi \frac{\pi-x}{2}\,dx=\frac{\pi^2}{4} \EA$

となるので
$\BA\D \zeta(2)=\frac{\pi^2}{6} \EA$

$Proof~6.$
$\BA\D \sum_{n=0}^\infty a_n^2=\L(\sum_{n=0}^\infty (-1)^na_n\R)^2+2\sum_{k=1}^\infty (-1)^{k-1}\sum_{n=0}^\infty a_na_{k+n} \EA$

において,$\D a_n=\frac{1}{2n+1}$とすると
右辺は

$\BA\D &\L(\sum_{n=0}^\infty \frac{(-1)^n}{2n+1}\R)^2+2\sum_{k=1}^\infty (-1)^{k-1}\sum_{n=0}^\infty \frac{1}{(2n+1)(2k+2n+1)}\\ =&\L(\frac{\pi}{4}\R)^2+2\sum_{k=1}^\infty \frac{(-1)^{k-1}}{2k}\sum_{m=0}^{k-1}\frac{1}{2m+1}\\ =&\frac{\pi^2}{16}+\sum_{k=1}^\infty \frac{(-1)^{k-1}}{k}\int_0^1 \frac{1-x^{2n}}{1-x^2}\,dx\\ =&\frac{\pi^2}{16}+\int_0^1 \frac{1}{1-x^2}\ln\frac{2}{1+x^2}\,dx\\ =&\frac{\pi^2}{16}+\int_0^1 \frac{\ln\frac{1}{x}}{1-x^2}\,dx-\int_0^1 \frac{1}{1-x^2}\ln\frac{1+x^2}{2x}\,dx\\ =&\frac{\pi^2}{16}+\frac{\pi^2}{8}-\int_0^1 \frac{1}{1-\L(\frac{1-t}{1+t}\R)^2}\ln\frac{1+\L(\frac{1-t}{1+t}\R)^2}{2\frac{1-t}{1+t}}\frac{2dt}{(1+t)^2}\\ =&\frac{3\pi^2}{16}-\int_0^1 \frac{\tanh^{-1}t^2}{t}\,dt\\ =&\frac{3\pi^2}{16}-\sum_{n=0}^\infty \frac{1}{2n+1} \int_0^1 t^{4n+1}dt\\ =&\frac{3\pi^2}{16}-\frac{1}{2}\sum_{n=0}^\infty \frac{1}{(2n+1)^2} \EA$

となり,左辺は
$\BA\D \sum_{n=0}^\infty \frac{1}{(2n+1)^2} \EA$

となるので
$\BA\D \sum_{n=0}^\infty \frac{1}{(2n+1)^2}=\frac{\pi^2}{8} \EA$

$Proof~7.$
$\BA\D \int_0^\frac{\pi}{2} x^2\cos^{2n-1}x\,dx=\frac{2^{2n}}{2n\binom{2n}{n}}\L(\frac{\pi^2}{8}-\sum_{k=0}^{n-1} \frac{1}{(2k+1)^2}\R) \EA$

が成り立ちます。これより明らかに

$\BA\D \frac{\pi^2}{8}-\sum_{k=0}^{n-1} \frac{1}{(2k+1)^2}>0 \EA$

また,
$\BA\D \sum_{k=0}^{n-1} \frac{1}{(2k+1)^2} &=\int_0^1 \frac{1-x^{2n}}{1-x^2}\ln\frac{1}{x}\,dx\\ &=\frac{1}{4}\int_0^1 \frac{1-x^n}{(1-x)\sqrt{x}}\ln\frac{1}{x}\,dx\\ &=\frac{1}{4}\int_0^1 \frac{1-x^n}{(1-x)\sqrt{x}}\L((1-x)+\frac{(1-x)^2}{2}+\cdots \R)dx\\ &>\frac{1}{4}\int_0^1 \frac{1-x^n}{(1-x)\sqrt{x}}\sum_{m=1}^M\frac{(1-x)^m}{m}\,dx\\ &=\sum_{m=1}^M\frac{1}{2m}\sum_{k=0}^{n-1}\frac{1}{2k+2m+1} \EA$

となります。ここで,前提として$n≫1$において
$\BA\D 2n\frac{\binom{2n}{n}}{2^{2n}}\int_0^\frac{\pi}{2}x^2\cos^{2n-1}x\,dx⪅\frac{1}{2n} \EA$

が成り立つことを考慮しつつ,これを証明します。すなわち,
$\BA\D \frac{\pi^2}{8}-\sum_{m=1}^M\frac{1}{2m}\sum_{k=0}^{n-1}\frac{1}{2k+2m+1}<\frac{1}{2n} \EA$

$n≫1$におい常に成り立つような$M$が存在することを示せばよいです。
実際に数値を確認すると,$M=2$の時点で,$n\ge 33$で不等式は成り立ちます。
よって,$n≫1$において
$\BA\D \frac{\pi^2}{8}-\sum_{k=0}^{n-1}\frac{1}{(2k+1)^2}<\frac{1}{2n} \EA$

が成り立つことがわかりました。すなわち
$\BA\D 0<\frac{\pi^2}{8}-\sum_{k=0}^{n-1}\frac{1}{(2k+1)^2}<\frac{1}{2n} \EA$

となり
$\BA\D \sum_{k=0}^\infty \frac{1}{(2k+1)^2}=\frac{\pi^2}{8} \EA$

となります。

$Proof~8.$
$\quad \sin $の無限乗積展開
$\BA\D \frac{\sin \pi x}{\pi x}=\prod_{n=1}^\infty \L(1-\frac{x^2}{n^2}\R) \EA$

と,$\rm Maclaurin~Series$

$\BA\D \frac{\sin \pi x}{\pi x}=\sum_{n=0}^\infty \frac{(-1)^{n}\pi^{2n}x^{2n}}{(2n+1)!} \EA$

$x^2$の係数を比較することで
$\BA\D \sum_{n=1}^\infty \frac{1}{n^2}=\frac{\pi^2}{6} \EA$

投稿日:2022120

この記事を高評価した人

高評価したユーザはいません

この記事に送られたバッジ

バッジはありません。

投稿者

コメント

他の人のコメント

コメントはありません。
読み込み中...
読み込み中